What are your chances of acceptance?

Calculate for all schools, your chance of acceptance.

Duke University

Your chancing factors

Extracurriculars.

ap lang argumentative essay

How to Write the AP Lang Argument Essay + Examples

What’s covered:, what is the ap language argument essay, tips for writing the ap language argument essay, ap english language argument essay examples, how will ap scores impact my college chances.

In 2023, over 550,148 students across the U.S. took the AP English Language and Composition Exam, and 65.2% scored higher than a 3. The AP English Language Exam tests your ability to analyze a piece of writing, synthesize information, write a rhetorical essay, and create a cohesive argument. In this post, we’ll be discussing the best way to approach the argumentative essay section of the test, and we’ll give you tips and tricks so you can write a great essay.

The AP English Language Exam as of 2023 is structured as follows:

Section 1: 45 multiple choice questions to be completed in an hour. This portion counts for 45% of your score. This section requires students to analyze a piece of literature. The questions ask about its content and/or what could be edited within the passage.

Section 2: Three free response questions to be completed in the remaining two hours and 15 minutes. This section counts for 55% of your score. These essay questions include the synthesis essay, the rhetorical essay, and the argumentative essay.

  • Synthesis essay: Read 6-7 sources and create an argument using at least three of the sources.
  • Rhetorical analysis essay: Describe how a piece of writing evokes meaning and symbolism.
  • Argumentative essay: Pick a side of a debate and create an argument based on evidence. In this essay, you should develop a logical argument in support of or against the given statement and provide ample evidence that supports your conclusion. Typically, a five paragraph format is great for this type of writing. This essay is scored holistically from 1 to 9 points.

Do you want more information on the structure of the full exam? Take a look at our in-depth overview of the AP Language and Composition Exam .

Although the AP Language Argument may seem daunting at first, once you understand how the essay should be structured, it will be a lot easier to create cohesive arguments.

Below are some tips to help you as you write the essay.

1. Organize your essay before writing

Instead of jumping right into your essay, plan out what you will say beforehand. It’s easiest to make a list of your arguments and write out what facts or evidence you will use to support each argument. In your outline, you can determine the best order for your arguments, especially if they build on each other or are chronological. Having a well-organized essay is crucial for success.

2. Pick one side of the argument, but acknowledge the other side

When you write the essay, it’s best if you pick one side of the debate and stick with it for the entire essay. All your evidence should be in support of that one side. However, in your introductory paragraph, as you introduce the debate, be sure to mention any merit the arguments of the other side has. This can make the essay a bit more nuanced and show that you did consider both sides before determining which one was better. Often, acknowledging another viewpoint then refuting it can make your essay stronger.

3. Provide evidence to support your claims

The AP readers will be looking for examples and evidence to support your argument. This doesn’t mean that you need to memorize a bunch of random facts before the exam. This just means that you should be able to provide concrete examples in support of your argument.

For example, if the essay topic is about whether the role of the media in society has been detrimental or not, and you argue that it has been, you may talk about the phenomenon of “fake news” during the 2016 presidential election.

AP readers are not looking for perfect examples, but they are looking to see if you can provide enough evidence to back your claim and make it easily understood.

4. Create a strong thesis statement

The thesis statement will set up your entire essay, so it’s important that it is focused and specific, and that it allows for the reader to understand your body paragraphs. Make sure your thesis statement is the very last sentence of your introductory paragraph. In this sentence, list out the key points you will be making in the essay in the same order that you will be writing them. Each new point you mention in your thesis should start a paragraph in your essay.

Below is a prompt and sample student essay from the May 2019 exam . We’ll look at what the student did well in their writing and where they could improve.

Prompt: “The term “overrated” is often used to diminish concepts, places, roles, etc. that the speaker believes do not deserve the prestige they commonly enjoy; for example, many writers have argued that success is overrated, a character in a novel by Anthony Burgess famously describes Rome as a “vastly overrated city,” and Queen Rania of Jordan herself has asserted that “[b]eing queen is overrated.”

Select a concept, place, role, etc. to which you believe that the term “overrated” should be applied. Then, write a well-developed essay in which you explain your judgment. Use appropriate evidence from your reading, experience, or observations to support your argument.

Sample Student Essay #1:

[1] Competition is “overrated.” The notion of motivation between peers has evolved into a source of unnecessary stress and even lack of morals. Whether it be in an academic environment or in the industry, this new idea of competition is harmful to those competing and those around them.

[2] Back in elementary school, competition was rather friendly. It could have been who could do the most pushups or who could get the most imaginary points in a classroom for a prize. If you couldn’t do the most pushups or win that smelly sticker, you would go home and improve yourself – there would be no strong feelings towards anyone, you would just focus on making yourself a better version of yourself. Then as high school rolled around, suddenly applying for college doesn’t seem so far away –GPA seems to be that one stat that defines you – extracurriculars seem to shape you – test scores seem to categorize you. Sleepless nights, studying for the next day’s exam, seem to become more and more frequent. Floating duck syndrome seems to surround you (FDS is where a competitive student pretends to not work hard but is furiously studying beneath the surface just like how a duck furiously kicks to stay afloat). All of your competitors appear to hope you fail – but in the end what do you and your competitor’s gain? Getting one extra point on the test? Does that self-satisfaction compensate for the tremendous amounts of acquired stress? This new type of “competition” is overrated – it serves nothing except a never-ending source of anxiety and seeks to weaken friendships and solidarity as a whole in the school setting.

[3] A similar idea of “competition” can be applied to business. On the most fundamental level, competition serves to be a beneficial regulator of prices and business models for both the business themselves and consumers. However, as businesses grew increasingly greedy and desperate, companies resorted to immoral tactics that only hurt their reputations and consumers as a whole. Whether it be McDonald’s coupons that force you to buy more food or tech companies like Apple intentionally slowing down your iPhone after 3 years to force you to upgrade to the newest device, consumers suffer and in turn speak down upon these companies. Similar to the evolved form of competition in school, this overrated form causes pain for all parties and has since diverged from the encouraging nature that the principle of competition was “founded” on.

The AP score for this essay was a 4/6, meaning that it captured the main purpose of the essay but there were still substantial parts missing. In this essay, the writer did a good job organizing the sections and making sure that their writing was in order according to the thesis statement. The essay first discusses how competition is harmful in elementary school and then discusses this topic in the context of business. This follows the chronological order of somebody’s life and flows nicely.

The arguments in this essay are problematic, as they do not provide enough examples of how exactly competition is overrated. The essay discusses the context in which competition is overrated but does not go far enough in explaining how this connects to the prompt.

In the first example, school stress is used to explain how competition manifests. This is a good starting point, but it does not talk about why competition is overrated; it simply mentions that competition can be unhealthy. The last sentence of that paragraph is the main point of the argument and should be expanded to discuss how the anxiety of school is overrated later on in life. 

In the second example, the writer discusses how competition can lead to harmful business practices, but again, this doesn’t reflect the reason this would be overrated. Is competition really overrated because Apple and McDonald’s force you to buy new products? This example could’ve been taken one step farther. Instead of explaining why business structures—such as monopolies—harm competition, the author should discuss how those particular structures are overrated.

Additionally, the examples the writer used lack detail. A stronger essay would’ve provided more in-depth examples. This essay seemed to mention examples only in passing without using them to defend the argument.

It should also be noted that the structure of the essay is incomplete. The introduction only has a thesis statement and no additional context. Also, there is no conclusion paragraph that sums up the essay. These missing components result in a 4/6.

Now let’s go through the prompt for a sample essay from the May 2022 exam . The prompt is as follows:

Colin Powell, a four-star general and former United States Secretary of State, wrote in his 1995 autobiography: “[W]e do not have the luxury of collecting information indefinitely. At some point, before we can have every possible fact in hand, we have to decide. The key is not to make quick decisions, but to make timely decisions.”

Write an essay that argues your position on the extent to which Powell’s claim about making decisions is valid. 

In your response you should do the following:

  • Respond to the prompt with a thesis that presents a defensible position. 
  • Provide evidence to support your line of reasoning. 
  • Explain how the evidence supports your line of reasoning. 
  • Use appropriate grammar and punctuation in communicating your argument.

Sample Student Essay #2:

Colin Powell, who was a four star general and a former United States Secretary of State. He wrote an autobiography and had made a claim about making decisions. In my personal opinion, Powell’s claim is true to full extent and shows an extremely valuable piece of advice that we do not consider when we make decisions.

Powell stated, “before we can have every possible fact in hand we have to decide…. but to make it a timely decision” (1995). With this statement Powell is telling the audience of his autobiography that it does not necessarily matter how many facts you have, and how many things you know. Being able to have access to everything possible takes a great amount of time and we don’t always have all of the time in the world. A decision has to be made with what you know, waiting for something else to come in while trying to make a decision whether that other fact is good or bad you already have a good amount of things that you know. Everyone’s time is valuable, including yours. At the end of the day the decision will have to be made and that is why it should be made in a “timely” manner.

This response was graded for a score of 2/6. Let’s break down the score to smaller points that signify where the student fell short.

The thesis in this essay is clearly outlined at the end of the first paragraph. The student states their agreement with Powell’s claim and frames the rest of their essay around this stance. The success in scoring here lies in the clear communication of the thesis and the direction the argument will take. It’s important to make the thesis statement concise, specific, and arguable, which the student has successfully done.

While the student did attempt to provide evidence to support their thesis, it’s clear that their explanation lacks specific detail and substance. They referenced Powell’s statement, but did not delve into how this statement has proven true in specific instances, and did not provide examples that could bring the argument to life.

Commentary is an essential part of this section’s score. It means explaining the significance of the evidence and connecting it back to the thesis. Unfortunately, the student’s commentary here is too vague and does not effectively elaborate on how the evidence supports their argument.

To improve, the student could use more concrete examples to demonstrate their point and discuss how each piece of evidence supports their thesis. For instance, they could discuss specific moments in Powell’s career where making a timely decision was more valuable than waiting for all possible facts. This would help illustrate the argument in a more engaging, understandable way.

A high score in the “sophistication” category of the grading rubric is given for demonstrating a complex understanding of the rhetorical situation (purpose, audience, context, etc.), making effective rhetorical choices, or establishing a line of reasoning. Here, the student’s response lacks complexity and sophistication. They’ve simply agreed with Powell’s claim and made a few general statements without providing a deeper analysis or effectively considering the rhetorical situation.

To increase sophistication, the student could explore possible counterarguments or complexities within Powell’s claim. They could discuss potential drawbacks of making decisions without all possible facts, or examine situations where timely decisions might not yield the best results. By acknowledging and refuting these potential counterarguments, they could add more depth to their analysis and showcase their understanding of the complexities involved in decision-making.

The student could also analyze why Powell, given his background and experiences, might have come to such a conclusion, thus providing more context and showing an understanding of the rhetorical situation.

Remember, sophistication in argumentation isn’t about using fancy words or complicated sentences. It’s about showing that you understand the complexity of the issue at hand and that you’re able to make thoughtful, nuanced arguments. Sophistication shows that you can think critically about the topic and make connections that aren’t immediately obvious.

Now that you’ve looked at an example essay and some tips for the argumentative essay, you know how to better prepare for the AP English Language and Composition Exam.

While your AP scores don’t usually impact your admissions chances , colleges do care a lot about your course rigor. So, taking as many APs as you can will certainly boost your chances! AP scores can be a way for high-performing students to set themselves apart, particularly when applying to prestigious universities. Through the process of self-reporting scores , you can show your hard work and intelligence to admissions counselors.

That said, the main benefit of scoring high on AP exams comes once you land at your dream school, as high scores can allow you to “test out” of entry-level requirements, often called GE requirements or distribution requirements. This will save you time and money.

To understand how your course rigor stacks up, check out CollegeVine’s free chancing engine . This resource takes your course rigor, test scores, extracurriculars, and more, to determine your chances of getting into over 1600 colleges across the country!

Related CollegeVine Blog Posts

ap lang argumentative essay

How to Write the AP Lang Argument Essay (With Example)

December 14, 2023

ap lang argument essay example

We’d like to let you in on a little secret: no one, including us, enjoys writing timed essays. But a little practice goes a long way. If you want to head into your AP English Exam with a cool head, you’ll want to know what you’re getting into ahead of time. We can’t promise the AP Lang Argument Essay will ever feel like an island vacation, but we do have tons of hand tips and tricks (plus a sample essay!) below to help you do your best. This article will cover: 1) What is the AP Lang Argumentative Essay? 2) AP Lang Argument Rubric 3) AP Lang Argument Sample Prompt 4) AP Lang Argument Essay Example 5) AP Lang Argument Essay Example: Answer Breakdown.

What is the AP Lang Argument Essay?

The AP Lang Argument Essay is one of three essays included in the written portion of the AP English Exam. The full AP English Exam is 3 hours and 15 minutes long, with the first 60 minutes dedicated to multiple-choice questions. Once you complete the multiple-choice section, you move on to three equally weighted essays that ask you to synthesize, analyze, and interpret texts and develop well-reasoned arguments. The three essays include:

Synthesis essay: You’ll review various pieces of evidence and then write an essay that synthesizes (aka combines and interprets) the evidence and presents a clear argument. Read our write-up on How to Write the AP Lang Synthesis Essay here.

Argumentative essay: You’ll take a stance on a specific topic and argue your case.

Rhetorical essay: You’ll read a provided passage, then analyze the author’s rhetorical choices and develop an argument that explains why the author made those rhetorical choices. Read our write-up on How to Write the AP Lang Rhetorical Essay here.

AP Lang Argument Essay Rubric

The AP Lang Argument Essay is graded on 3 rubric categories : Thesis, Evidence and Commentary, and Sophistication . How can you make sure you cover all three bases in your essay? We’ll break down each rubric category with dos and don’ts below:

  • Thesis (0-1 point)

When it comes to grading your thesis, AP Exam graders are checking off a box: you either have a clear thesis or you don’t. So, what crucial components of a thesis will get you your check mark?

  • Make sure your thesis argues something . To satisfy your graders, your thesis needs to take a clear stance on the issue at hand.
  • Include your thesis statement in your intro paragraph. The AP Lang Argumentative essay is just that: an essay that makes an argument, so make sure you present your argument right away at the end of your first paragraph.
  • A good test to see if you have a thesis that makes an argument for your AP Lang Argumentative Essay: In your head, add the phrase “I agree/disagree that…” to the beginning of your thesis. If what follows doesn’t logically flow after that phrase (aka if what follows isn’t an agreement or disagreement), it’s likely you’re not making an argument.
  • In your thesis, outline the evidence you’ll cover in your body paragraphs.

AP Lang Argument Essay Rubric (Continued)

  • Avoid a thesis that merely restates the prompt.
  • Avoid a thesis that summarizes the text but does not make an argument.
  • Avoid a thesis that weighs the pros and cons of an issue. Your job in your thesis is to pick a side and stick with it.
  • Evidence and Commentary (0-4 points)

This rubric category is graded on a scale of 0-4 where 4 is the highest grade. Unlike the rhetorical and synthesis essays, the evidence you need to write your AP Lang Argument Essay is not provided to you. Rather, you’ll need to generate your own evidence and comment upon it.

What counts as evidence?

Typically, the AP Lang Argument Essay prompt asks you to reflect on a broad cultural, moral, or social issue that is open to debate. For evidence, you won’t be asked to memorize and cite statistics or facts. Rather, you’ll want to bring in real-world examples of:

  • Historical events
  • Current-day events from the news
  • Personal anecdotes

For this essay, your graders know that you’re not able to do research to find the perfect evidence. What’s most important is that you find evidence that logically supports your argument.

What is commentary?

In this essay, it’s important to do more than just provide examples relevant evidence. After each piece of evidence you include, you’ll need to explain why it’s significant and how it connects to your main argument. The analysis you include after your evidence is commentary .

  • Take a minute to brainstorm evidence that logically supports your argument. If you have to go out of your way to find the connection, it’s better to think of different evidence.
  • Include multiple pieces of evidence. There is no magic number, but do make sure you incorporate more than a couple pieces of evidence that support your argument.
  • Make sure you include more than one example of evidence, too. Let’s say you’re working on an essay that argues that people are always stronger together than apart. You’ve already included an example from history: during the civil rights era, protestors staged group sit-ins as a powerful form of peaceful protest. That’s just one example, and it’s hard to make a credible argument with just one piece of evidence. To fix that issue, think of additional examples from history, current events, or personal experience that are not related to the civil rights era.
  • After you include each piece of evidence, explain why it’s significant and how it connects to your main argument.
  • Don’t summarize or speak generally about the topic. Everything you write must be backed up with specific and relevant evidence and examples.
  • Don’t let quotes speak for themselves. After every piece of evidence you include, make sure to explain and connect the evidence to your overarching argument.

AP Lang Argument Essay (Continued)

  • Sophistication (0-1 point)

According to the College Board , one point can be awarded to AP Lang Argument essays that achieve a high level of sophistication. You can accomplish that in four ways:

  • Crafting a nuanced argument by consistently identifying and exploring complexities or tensions.
  • Articulating the implications or limitations of an argument by situating it within a broader context.
  • Making effective rhetorical choices that consistently strengthen the force and impact of the student’s argument.
  • Employing a style that is consistently vivid and persuasive.

In sum, this means you can earn an additional point for going above and beyond in depth, complexity of thought, or by writing an especially persuasive, clear, and well-structured essay. In order to earn this point, you’ll first need to do a good job with the fundamentals: your thesis, evidence, and commentary. Then, to earn your sophistication point, follow these tips:

  • Outline your essay before you begin to ensure it flows in a clear and cohesive way.
  • Include well-rounded evidence. Don’t rely entirely on personal anecdotes, for example. Incorporate examples from current events or history, as well.
  • Thoroughly explain how each piece of evidence connects to your thesis in order to fully develop your argument.
  • Explore broader implications. If what you’re arguing is true, what does that mean to us today? Who is impacted by this issue? What real-world issues are relevant to this core issue?
  • Briefly explore the other side of the issue. Are the instances where your argument might not be true? Acknowledge the other side, then return to proving your original argument.
  • Steer clear of generalizations (avoid words like “always” and “everyone”).
  • Don’t choose an argument you can’t back up with relevant examples.
  • Avoid complex sentences and fancy vocabulary words unless you use them often. Long, clunky sentences with imprecisely used words are hard to follow.

AP Lang Argument Sample Prompt

The sample prompt below is published online by the College Board and is a real example from the 2021 AP English Exam. The prompt provides background context, essay instructions, and the text you need to analyze.

Suggested time—40 minutes.

Many people spend long hours trying to achieve perfection in their personal or professional lives. Similarly, people often demand perfection from others, creating expectations that may be challenging to live up to. In contrast, some people think perfection is not attainable or desirable.

Write an essay that argues your position on the value of striving for perfection.

In your response you should do the following:

  • Respond to the prompt with a thesis that presents a defensible position.
  • Provide evidence to support your line of reasoning.
  • Explain how the evidence supports your line of reasoning.
  • Use appropriate grammar and punctuation in communicating your argument.

AP Lang Argument Essay Example

As the old phrase says, “Practice makes perfect.” But is perfection something that is actually attainable? Sometimes, pushing for perfection helps us achieve great things, but most often, perfectionism puts too much pressure on us and prevents us from knowing when we have done the best we can. Striving for perfection can only lead us to shortchange ourselves. Instead, we should value learning, growth, and creativity and not worry whether we are first or fifth best.

Students often feel the need to be perfect in their classes, and this can cause students to struggle or stop making an effort in class. In elementary and middle school, for example, I was very nervous about public speaking. When I had to give a speech, my voice would shake, and I would turn very red. My teachers always told me “relax!” and I got Bs on Cs on my speeches. As a result, I put more pressure on myself to do well, spending extra time making my speeches perfect and rehearsing late at night at home. But this pressure only made me more nervous, and I started getting stomach aches before speaking in public.

Once I got to high school, however, I started doing YouTube make-up tutorials with a friend. We made videos just for fun, and laughed when we made mistakes or said something silly. Only then, when I wasn’t striving to be perfect, did I get more comfortable with public speaking.

AP Lang Argumentative Essay Example (Continued)

In the world of art and business and science, perfectionism can also limit what we are able to achieve. Artists, for example, have to take risks and leave room for creativity. If artists strive for perfection, then they won’t be willing to fail at new experiments and their work will be less innovative and interesting. In business and science, many products, like penicillin for example, were discovered by accident. If the scientist who discovered penicillin mold growing on his petri dishes had gotten angry at his mistake and thrown the dishes away, he would never have discovered a medicine that is vital to us today.

Some fields do need to value perfection. We wouldn’t like it, for example, if our surgeon wasn’t striving for perfection during our operation. However, for most of us, perfectionism can limit our potential for learning and growth. Instead of trying to be perfect, we should strive to learn, innovate, and do our personal best.

AP Lang Argument Essay Example: Answer Breakdown

The sample AP Lang Argumentative Essay above has some strengths and some weaknesses. Overall, we would give this essay a 3 or a 4. Let’s break down what’s working and what could be improved:

  • The essay offers a thesis that makes a clear argument that is relevant to the prompt: “Striving for perfection can only lead us to shortchange ourselves. Instead, we should value learning, growth, and creativity and not worry whether we are first or fifth best.”
  • The first body paragraph provides evidence that supports the essay’s thesis. This student’s personal anecdote offers an example of a time when perfectionism led them to shortchange themselves.
  • The second body paragraph provides additional evidence that supports the essay’s thesis. The example describing the discovery of penicillin offers another example of a situation in which perfectionism might have limited scientific progress.
  • The writer offers commentary explaining how her examples of public speaking and penicillin illustrate that we should “value learning, growth, and creativity” over perfectionism.
  • The essay follows one line of reasoning and does not stray into tangents.
  • The essay is organized well with intro, body, and concluding paragraphs. Overall, it is easy to read and is free of grammar errors.

What could be improved:

  • Although the second body paragraph provides one good specific example about the discovery of penicillin, the other examples it offers about art and business are only discussed generally and aren’t backed up with evidence. This paragraph would be stronger if it provided more examples. Or, if this writer couldn’t think of examples, they could have left out mentions of art and business altogether and included alternate evidence instead.
  • This writer would more thoroughly support their argument if they were able to offer one more example of evidence. They could provide another personal anecdote, an example from history, or an example from current events.
  • The writer briefly mentions the other side of the argument in their concluding paragraph: “Some fields do need to value perfection. We wouldn’t like it, for example, if our surgeon wasn’t striving for perfection during our operation.” Since it’s so brief a mention of the other side, it undermines the writer’s overall argument. This writer should either dedicate more time to reflecting on why even surgeons should “value learning, growth, and creativity” over perfectionism, or they should leave these sentences out.

AP Lang Argument Essay Example—More Resources

Looking for more tips to help you master your AP Lang Argumentative Essay? Brush up on 20 Rhetorical Devices High School Students Should Know and read our Tips for Improving Reading Comprehension .

If you’re ready to start studying for another part of the AP English Exam, find more expert tips in our How to Write the AP Lang Synthesis and How to Write the AP Lang Rhetorical Essay blog posts.

  • High School Success

' src=

Christina Wood

Christina Wood holds a BA in Literature & Writing from UC San Diego, an MFA in Creative Writing from Washington University in St. Louis, and is currently a Doctoral Candidate in English at the University of Georgia, where she teaches creative writing and first-year composition courses. Christina has published fiction and nonfiction in numerous publications, including The Paris Review , McSweeney’s , Granta , Virginia Quarterly Review , The Sewanee Review , Mississippi Review , and Puerto del Sol , among others. Her story “The Astronaut” won the 2018 Shirley Jackson Award for short fiction and received a “Distinguished Stories” mention in the 2019 Best American Short Stories anthology.

  • 2-Year Colleges
  • Application Strategies
  • Big Picture
  • Career & Personality Assessment
  • College Essay
  • College Search/Knowledge
  • College Success
  • Costs & Financial Aid
  • Dental School Admissions
  • Extracurricular Activities
  • Graduate School Admissions
  • High Schools
  • Law School Admissions
  • Medical School Admissions
  • Navigating the Admissions Process
  • Online Learning
  • Private High School Spotlight
  • Summer Program Spotlight
  • Summer Programs
  • Test Prep Provider Spotlight

College Transitions Sidebar Block Image

“Innovative and invaluable…use this book as your college lifeline.”

— Lynn O'Shaughnessy

Nationally Recognized College Expert

College Planning in Your Inbox

Join our information-packed monthly newsletter.

Sign Up Now

ap lang argumentative essay

Crafting an Impressive Argumentative Essay for AP Lang

ap lang argumentative essay

Writing an argumentative essay for AP Language and Composition requires a strategic approach to effectively convey your perspective. Here's a guide to crafting an impressive argumentative essay:

1. Understand the Prompt:

   - Tip: Carefully read and analyze the prompt. Identify the key elements, including the task you are asked to perform and any specific requirements or constraints.

2. Develop a Clear Thesis Statement:

   - Tip: Formulate a concise and focused thesis statement that clearly states your argument or position. This statement should guide the reader on what to expect in your essay.

3. Identify Your Audience:

   - Tip: Consider your target audience and tailor your language and arguments accordingly. Understanding your audience helps you establish a connection and effectively convey your message.

4. Organize Your Essay Effectively:

   - Tip: Structure your essay with a clear introduction, body paragraphs, and a conclusion. Ensure a logical flow of ideas, with each paragraph contributing to the overall argument.

5. Provide Context and Background:

   - Tip: Begin with a brief introduction that provides context for your argument. Clearly state the issue at hand and offer background information to help readers understand the significance of your argument.

6. Present a Strong Claim:

   - Tip: Clearly articulate your main claim or argument. This should be a debatable statement that forms the core of your essay. Avoid vague or overly broad claims.

7. Support Your Argument with Evidence:

   - Tip: Use relevant and compelling evidence to support your claims. This can include facts, statistics, examples, anecdotes, or quotations. Ensure that your evidence is credible and directly contributes to your argument.

8. Address Counterarguments:

   - Tip: Acknowledge potential counterarguments and address them in your essay. This demonstrates a nuanced understanding of the issue and strengthens your overall argument.

9. Use Persuasive Language:

    - Tip: Choose language that is persuasive and impactful. Utilize rhetorical devices, vivid imagery, and compelling language to engage your readers and convey the urgency or importance of your argument.

10. Craft Well-Reasoned Body Paragraphs:

    - Tip: Dedicate separate paragraphs to different aspects of your argument. Each paragraph should have a clear topic sentence, supporting evidence, and a concise explanation of how the evidence relates to your thesis.

11. Ensure Cohesive Transitions:

    - Tip: Use transitional phrases and sentences to ensure a smooth transition between paragraphs. This enhances the coherence of your essay and guides readers through your line of reasoning.

12. Conclude Effectively:

    - Tip: Summarize your main points in the conclusion and restate the significance of your argument. Avoid introducing new information in the conclusion. End with a strong closing statement that leaves a lasting impression.

13. Revise and Edit:

    - Tip: Set aside time for revision and editing. Review your essay for clarity, coherence, and grammatical accuracy. Ensure that your argument is compelling and well-supported.

14. Seek Feedback:

    - Tip: If possible, seek feedback from peers, teachers, or mentors. External perspectives can provide valuable insights and help you identify areas for improvement.

15. Stay Focused on the Argument:

    - Tip: Throughout the essay, stay focused on your central argument. Avoid unnecessary tangents or diverging from the main point. Every part of your essay should contribute to strengthening your overall argument.

Crafting an impressive argumentative essay for AP Language and Composition involves a combination of persuasive writing techniques, solid evidence, and effective organization. By following these tips, you can create a compelling essay that showcases your ability to engage in sophisticated argumentation and rhetorical analysis.

You Might Also Like

ap lang argumentative essay

How to Write a Recommendation Letter for College Admissions

Learn some tips that you can do to ensure that your recommendation letter get accepted and you can get admission in your dream university/college - Read a blog

ap lang argumentative essay

How can Conducting Research get you into Your Dream College

Want to get admission in your dream college? Do formal research for college admission that will help you to gain admission in your dream college - Read a blog

ap lang argumentative essay

Scholarship Application Process

Filling out college scholarship applications is indeed a time-consuming task that needs much effort and patience. This guide will help you through the entire scholarship application process

AP Guru has been helping students since 2010 gain admissions to their dream universities by helping them in their college admissions and SAT and ACT Prep

Free Resources

logo-type-white

AP® English Language

How to craft an argument for ap® english language.

  • The Albert Team
  • Last Updated On: March 1, 2022

how_to_craft_an_argument_for_ap_english_language

The AP® English Language persuasive (or argumentative) essay is one of the three long-form free-response questions that will make up 55% of your score on the AP® English Language and Composition Exam. While the multiple-choice section and the rhetorical analysis essay will test you on how well you have learned the various rhetorical techniques you have been exposed to this year, the persuasive essay and a similar task, the synthesis (also see our article “5 Tips to the AP® English Language Synthesis Essay You Must Know”), will test you on how well you can put these techniques to use yourself.

It’s time for you to follow in the footsteps of the established, respected writers you have been reading all year and put everything that you’ve learned to work in the AP® English Language persuasive essay.

How the AP® English Language Persuasive Essay Works

Persuasion through essay writing is something you probably learned about a long time ago, but the AP® English Language Exam’s persuasion essay requires some more specific tips. You will be given a prompt that may or may not reference a reading sample; it will ask you to then “defend,” “challenge,” or “qualify” a position on a public issue – either the position espoused in the reading sample or one simply stated by the author of the question.

To defend a position is to agree with it and rationalize that agreement, to challenge it is to disagree with it and show holes in its supporting logic. To qualify a position is to attempt to truly understand all sides of the issue and see that both sides may have some valid points. However, you still need to take a definite stand, no matter what you do, although it can be a stand such as “Idea X is ethical in certain situations and unethical in others” – however, expand on that to give the AP® Examiners an exact notion of your opinion, and then use logic and beautiful writing to persuade them to see your way of thinking.

No Issue is One-Sided

Although taking a definitive stand is one of the most important things you need to do during the AP® English Language persuasive essay, you will often score higher if you show the full complexity of issues and exhibit understanding of the other side of the argument. This can not only show that you are intelligent and appreciate the complexity of the types of issues you may be talking about on the exam, but may actually help strengthen your argument, in that you can foresee potential arguments against your support for your beliefs, then undermine them as you write about them.

Even in issues that you are very passionate about or cannot see the other side’s logic on at all, keep in mind that you should be respectful and mature in all your AP® Exam writings.

Draw from All Possible Sources – But Don’t Be Self-Centered!

This AP® Language persuasive essay allows you to draw on your knowledge from other subjects, what you’ve read inside and outside of school (be it a classic novel or this morning’s paper), and your personal experience; a well-rounded, well-thought-out essay will use all or at least most of these. That being said, don’t be too focused on using your own experience to justify your beliefs – this is a less mature, less powerfully logical way of arguing than what the Examiners expect. Use personal experience, when relevant, as one facet of a wider, more nationally and globally aware argument.

For example, a prompt on advertising could probably use some personal anecdotes about your experiences with advertising alongside things you may have seen in the news or learned in a statistics class and analogies you can draw using global events or literature. A prompt on the ethics of experimentation on animals probably shouldn’t use much personal experience (unless you have a biologist in the family), because your “experiences” will be limited to feelings, not fully lived and understood events that will hold up in an argument.

Looking for AP® English Language practice?

Kickstart your AP® English Language prep with Albert. Start your AP® exam prep today .

Interested in a school license?​

Popular posts.

AP® Physics I score calculator

AP® Score Calculators

Simulate how different MCQ and FRQ scores translate into AP® scores

ap lang argumentative essay

AP® Review Guides

The ultimate review guides for AP® subjects to help you plan and structure your prep.

ap lang argumentative essay

Core Subject Review Guides

Review the most important topics in Physics and Algebra 1 .

ap lang argumentative essay

SAT® Score Calculator

See how scores on each section impacts your overall SAT® score

ap lang argumentative essay

ACT® Score Calculator

See how scores on each section impacts your overall ACT® score

ap lang argumentative essay

Grammar Review Hub

Comprehensive review of grammar skills

ap lang argumentative essay

AP® Posters

Download updated posters summarizing the main topics and structure for each AP® exam.

Interested in a school license?

ap lang argumentative essay

Bring Albert to your school and empower all teachers with the world's best question bank for: ➜ SAT® & ACT® ➜ AP® ➜ ELA, Math, Science, & Social Studies aligned to state standards ➜ State assessments Options for teachers, schools, and districts.

The Study Corp Logo

Mastering the Argumentative Essay in AP Lang: Strategies and Examples

  • December 8, 2023
  • Study Guides

In AP Language, crafting an argumentative essay requires specific skills and strategies. Students often struggle with providing credible evidence and effectively defending, challenging, or qualifying their claims. To excel in the argumentative essay, it is crucial for students to carefully read and follow the prompt instructions. Prompts can vary, but they typically ask students to defend, challenge, or qualify assertions on various topics. The use of appropriate evidence is emphasized, with prompts instructing students to support their arguments with specific references to reading, observation, or experience.

Here's What You'll Learn

To create a strong argumentative essay, students should follow certain strategies. Firstly, they need to clearly define their thesis or main argument and ensure it is rooted in evidence. Secondly, developing a well-structured essay with a logical flow of ideas is essential. Each body paragraph should present a separate piece of evidence and provide analysis to support the thesis. Students should also consider counterarguments and address them within the essay. Incorporating persuasive techniques such as ethos, pathos, and logos can enhance the effectiveness of the essay. Lastly, allocating enough time for revision and editing is crucial to ensure clarity and coherence in their writing.

Key Takeaways:

  • Carefully read and follow the prompt instructions
  • Clearly define the thesis and ensure it is supported by evidence
  • Develop a well-structured essay with a logical flow of ideas
  • Consider counterarguments and address them within the essay
  • Incorporate persuasive techniques like ethos, pathos, and logos

By understanding the prompt, developing a clear thesis, providing relevant evidence, and utilizing persuasive techniques , students can create compelling argumentative essays in AP Lang. With practice and dedication, their writing skills will improve, leading to success in the AP Lang exam.

Strategies for Crafting a Strong Argumentative Essay

To create a strong argumentative essay, students should follow certain strategies. First, they need to clearly define their thesis or main argument and ensure it is rooted in evidence. The thesis statement should be concise and provide direction for the essay. Next, students should develop a well-structured essay with a logical flow of ideas. Each body paragraph should present a separate piece of evidence and provide analysis to support the thesis.

It is also important for students to consider counterarguments and address them within the essay. This shows a deeper understanding of the topic and strengthens the overall argument. In addition, incorporating persuasive techniques such as ethos, pathos, and logos can enhance the effectiveness of the essay. These techniques appeal to the reader’s emotions, logic, and credibility, respectively, making the argument more persuasive and convincing.

Finally, students should allocate enough time for revision and editing to ensure clarity and coherence in their writing. By reviewing and refining their essay, students can eliminate any inconsistencies or ambiguities and make their argument more compelling. Taking the time to revise also allows students to improve their use of language, grammar, and punctuation, resulting in a polished and professional essay.

Examples of Strong Argumentative Essays in AP Lang

To provide students with a clear understanding of what constitutes a strong argumentative essay in AP Lang, it is helpful to provide examples. One example is an essay that received a high score in the exam. The essay started with a strong introduction that grabbed the reader’s attention and clearly stated the thesis. The body paragraphs presented multiple pieces of evidence that were carefully analyzed and connected back to the thesis. The essay also made concessions to the opposing side, showing a balanced perspective. The conclusion effectively summarized the main points and restated the thesis in a compelling manner.

Another example is an essay that effectively used persuasive techniques to enhance the argument. The use of emotional appeals, logical reasoning, and credibility helped convince the reader of the essay’s validity. These examples demonstrate the skills and strategies needed to craft a successful argumentative essay in AP Lang.

These examples serve as inspiration for students aspiring to become top essay writers . By studying and analyzing these successful essays, students can gain insights into effective argumentative writing strategies. It is important to note that crafting such essays requires practice and dedication. Professional paper writers have honed their skills through years of experience, making them valuable resources for students seeking guidance. With the right support and practice, students can develop their own abilities as pro essay writers and excel in the AP Lang exam.

Crafting an AP Lang argumentative essay requires a combination of skills, strategies, and practice. By understanding the prompt, developing a clear thesis, providing relevant evidence, and utilizing persuasive techniques, students can create compelling essays. It is important to allocate enough time for preparation, revision, and editing to ensure a well-structured and coherent essay.

With dedicated study and practice, students can improve their argumentative writing skills and achieve success in the AP Lang exam. And if you’re looking for reliable dissertation writers or professional paper writers to guide and support you in crafting an effective AP Lang argumentative essay, look no further. At The Study Corp, we have a team of experts ready to assist you in your academic journey.

Our reliable dissertation writers and professional paper writers have years of experience in writing and can help you develop strong arguments, provide expert analysis, and deliver high-quality essays. Don’t let writing assignments overwhelm you – let us be your academic partner and help you excel in your AP Lang essays.

What skills are necessary to excel in the argumentative essay in AP Lang?

To excel in the argumentative essay, students must develop their analysis skills, understand the prompt, and provide relevant and convincing evidence to support their claims.

What strategies should students follow to create a strong argumentative essay in AP Lang?

Students should clearly define their thesis, develop a well-structured essay with a logical flow of ideas, consider counterarguments, incorporate persuasive techniques, and allocate enough time for revision and editing.

Can you provide examples of strong argumentative essays in AP Lang?

Yes, examples of strong argumentative essays include those that have a strong introduction, well-analyzed evidence, concessions to the opposing side, effective conclusions, and persuasive techniques that enhance the argument.

How can students improve their argumentative writing skills in AP Lang?

By understanding the prompt, developing a clear thesis, providing relevant evidence, utilizing persuasive techniques, and dedicating time for preparation, revision, and editing, students can improve their argumentative writing skills in AP Lang.

Source Links

  • https://www.albert.io/blog/how-to-get-a-6-on-argument-frq-in-ap-english-language/
  • https://www.albert.io/blog/master-ap-arguing/
  • https://www.georgetownisd.org/cms/lib/TX01001838/Centricity/Domain/594/Argument essay.pdf

Does this Look Like Your Assignment? We Can do an Original Paper for you!

Have no time to write let a subject expert write your paper for you​, get writing assistance, worried about your paper we can help, have a subject expert write for you, find essays, papers….

Essay Topics and Ideas (99) Sample Study Papers (1487) Study Guides (432) Writing Guides (10)

Academic Success and Professional Development Plan (4) argumentative essay topics (2) Arizona State University (10) Capella University (58) Chamberlain University (40) DNP Assignments (76) Essay Topics (13) Grand Canyon University (77) Herzing University (38) Ideas (15) Management Assignments (7) Management Assignments Help (6) Management Guides (6) MSN Assignments (65) nursing care plans (20) Nursing Care Plans Examples (20) Solved essays (237) Student's Essays (12) thestudycorp.com (32) Topics (15) Topics, Ideas (38) Walden University (78)

  • Company Overview
  • Our Guarantees
  • Client Reviews
  • Discount Codes
  • Privacy Policy
  • Contact Us 
  • Terms and Conditions
  • Cookie Policy
  • Frequently Asked Questions
  • Fair Use Policy
  • Revisions and Refund Policy

Knowledge Base

  • All Writing Guides 
  • Nursing Essay Writing Guides 
  • Topics Ideas
  • Nursing Guides
  • Business Analysis Guides
  • Literature Guides 
  • Write My Essay 
  • Do My Essay
  • Pay For Essay
  • Buy Research Paper 
  • Buy Essays 
  • Get Nursing Papers
  • Online Nursing Papers

Writing Tools

  • Citation Generator
  • Topic Generator
  • Thesis Generator
  • Sentence Rewriter
  • Title Page Generator
  • Research Paper Title Generator

Use our resources and guides to write perfect papers. You can use our writing service and order customized sample papers without plagiarism!

Thestudycorp.com helps students cope with college assignments and write papers on various topics. We deal with academic writing, creative writing, and non-word assignments.

All the materials from our website should be used with proper references. All the work should be used per the appropriate policies and applicable laws.

Our samples and other types of content are meant for research and reference purposes only. We are strongly against plagiarism and academic dishonesty.

ap lang argumentative essay

Fiveable

Find what you need to study

2024 AP English Language and Composition Exam Guide

12 min read • august 18, 2023

A Q

Your guide to the 2024 AP English Language and Composition exam

We know that studying for your AP exams can be stressful, but Fiveable has your back! We created a study plan to help you crush your AP English Language and Composition exam. This guide will continue to update with information about the 2024 exams, as well as helpful resources to help you do your best on test day.  Unlock Cram Mode  for access to our cram events—students who have successfully passed their AP exams will answer your questions and guide your last-minute studying LIVE! And don't miss out on unlimited access to our database of thousands of practice questions. FYI, something cool is coming your way Fall 2023! 👀

Format of the 2024 AP English Language and Composition exam

This year, all AP exams will cover all units and essay types. The 2024 AP English Language and Composition exam format will be:

Section I: Multiple Choice - 45% of your score

45 questions in 1 hour

Section II: Free Response Section - 55% of your score

2 hours and 15 minutes for:

1 synthesis essay

1 rhetorical analysis essay

1 argument essay

Scoring Rubric for the 2024 AP Lang Essays

Synthesis Essay

1 point for a defensible thesis that responds to the prompt

Evidence and Commentary

Max of 4 points for providing evidence from at least 3 sources that support the line of reasoning AND commentary that explains and analyzes the evidence

Sophistication

1 point any of the following:

Creating a nuanced argument

Showing the limitations of the argument

Making effective rhetorical choices

Employing a style that is vivid and persuasive

Rhetorical Analysis Essay

1 point for a defensible thesis that analyzes rhetorical choices

Max of 4 points for providing specific evidence AND consistently explaining how the evidence relates to the line of reasoning AND showing how the rhetorical choices contribute to the author's message .

1 point for any of the following:

Explaining the significance of the rhetorical choices ( rhetorical situation )

Explaining the complexities of the passage and their purpose

Argument Essay

1 point for a defensible thesis

Max of 4 points for providing specific evidence AND consistently explaining the relevance of that evidence .

Crafting a nuanced argument by identifying complexities

Explaining the limitations of the argument by placing it in a broader context

Making rhetorical choices to improve the argument

Check out our study plan below to find resources and tools to prepare for your AP English Language and Composition exam.

When is the 2024 AP English Language and Composition Exam and How Do I Take It?

How should i prepare for the ap lang exam.

First, take stock of your progress in the course so far. What areas have you excelled and which sections need more focus? Download the AP English Language Cheatsheet PDF - a single sheet that covers everything you need to know at a high level. Take note of your strengths and weaknesses!

Build your study plan to review every unit and question type, but focus most on the areas that need the most improvement and practice. We’ve put together this plan to help you study between now and May. This will cover all of the units and essay types to prepare you for your exam

Practice essays are your best friends! The more essays you write, the more automatic the process will come, and the easier the AP exam will be!

Try some of the past exam questions here

We've put together the study plan found below to help you study between now and May. This will cover all of the units and essay types to prepare you for your exam. Pay special attention to the units that you need the most improvement in.

Study, practice, and review for test day with other students during our live cram sessions via  Cram Mode . Cram live streams will teach, review, and practice important topics from AP courses, college admission tests, and college admission topics. These streams are hosted by experienced students who know what you need to succeed.

Pre-Work: Set Up Your Study Environment

Before you begin studying, take some time to get organized.

🖥 Create a study space.

Make sure you have a designated place at home to study. Somewhere you can keep all of your materials, where you can focus on learning, and where you are comfortable. Spend some time prepping the space with everything you need and you can even let others in the family know that this is your study space. 

📚 Organize your study materials.

Get your notebook, textbook, prep books, or whatever other physical materials you have. Also, create a space for you to keep track of review. Start a new section in your notebook to take notes or start a Google Doc to keep track of your notes. Get yourself set up!

📅 Plan designated times for studying.

The hardest part about studying from home is sticking to a routine. Decide on one hour every day that you can dedicate to studying. This can be any time of the day, whatever works best for you. Set a timer on your phone for that time and really try to stick to it. The routine will help you stay on track.

🏆 Decide on an accountability plan.

How will you hold yourself accountable to this study plan? You may or may not have a teacher or rules set up to help you stay on track, so you need to set some for yourself. First, set your goal. This could be studying for x number of hours or getting through a unit. Then, create a reward for yourself. If you reach your goal, then x. This will help stay focused!

🤝 Get support from your peers.  

There are thousands of students all over the world who are preparing for their AP exams just like you! Join  Rooms  🤝 to chat, ask questions, and meet other students who are also studying for the spring exams. You can even build study groups and review material together! 

2024 AP Lang Study Guide

🚧 unit 1 foundations of rhetoric: analysis of the rhetorical situation and claims ., big takeaways:.

Unit 1 is an introductory unit that lays the foundations for the reading skills associated with how to understand and analyze complex texts. Skills here include identifying the ASPECTS of a text, analyzing the claim given and the evidence used to support that claim, and determining the function of the “chunks” in the argument. Because the content in this unit is very foundational, it is looped throughout the rest of the course instruction.

Definitely do this:

📚 Read these study guides:

Unit 1 Overview: Claims , Reasoning , and Evidence

1.1 Identifying the purpose and intended audience of a text

1.2 Examining how evidence supports a claim

1.3 Developing paragraphs as part of an effective argument

🎥 Watch these videos:

College Board’s Instructional Video: Overview of The Rhetorical Situation .

Fiveable’s How to Read Like an AP Student .

Rhetorical Analysis Thesis Statements  

Rhetorical Analysis Body Paragraphs

✍️ Practice:

Use the Fiveable ASPECTS Guidesheet to help you break down a complex text.

🗺 Can you identify these rhetorical devices?

You won’t be asked to name drop on the exam, but it can be helpful to use devices when discussing strategies. Try this Quizlet to help prepare.

Unit 2 Foundations of Argument: Analysis of an author’s choices in appeals and evidence

Unit 2 is an introductory unit that builds onto the foundations of rhetorical ASPECTS and moves toward planning and writing your own arguments. This unit focuses on the relationships between subject, speaker, and message, including examination of the structure and purpose of the given argument. The unit then moves into the developing thesis statements and building your own arguments with a clear line of reasoning .

Unit 2 Overview: Organizing Information for a Specific Audience

2.1 Analyzing audience and its relationship to the purpose of an argument

2.2 Building an argument with relevant and strategic evidence

2.3 Developing thesis statements

2.4 Developing structure and integrating evidence to reflect a line of reasoning

College Board’s Instructional Video: Identify Rhetorical Situation in a Pre 20th Century Text .

Fiveable’s video on How to Find Rhetorical Devices  

📰 Check out these articles:

Here’s a list of recommended rhetorical devices with definitions and examples!

Use the Fiveable Rhetorical Precis Guidesheet to help you break down a complex text.

🗺 Can you identify these elements of practical argument?

You won’t be asked to name drop of the exam, but it can be helpful to use devices when discussing strategies. Try this Quizlet to help prepare.

👥 Unit 3 Confluence: Synthesis of multiple sources in argumentation

Unit 3 approaches multiple perspectives in argument through the lens of synthesis (that’s FRQ 1). In this study, you learn to identify effective and faulty reasoning while integrating a variety of evidence from credible resources that is properly cited in an original text.

Unit 3 Overview: Perspectives and How Arguments Relate

3.1 Interpreting character description and perspective

3.2 Identifying and avoiding flawed lines of reasoning

3.3 Introducing and integrating sources and evidence

3.4 Using sufficient evidence for an argument

3.5 Attributing and citing references

3.6 Developing parts of a text with cause-effect and narrative methods

Fiveable’s Introduction into Synthesis Essays and How to Begin Your Argument

College Board’s Instructional Video: Complexity in Argument .

🗺 Can you identify these elements of synthesis?

👀 Unit 4 Reasoning : Analysis of argument from introduction to conclusion

Unit 4 includes a greater depth of focus on the writing of effective arguments -- the line of reasoning created in the introduction, built with modes of discourse, and strengthened in the conclusion. An important note about these skills of argumentation is that they build toward all parts of every FRQ. 

Unit 4 Overview: How writers develop arguments, intros, and conclusion

4.1 Developing and connecting thesis statements and lines of reasoning

4.2 Developing introductions and conclusions

4.3 Adjusting an argument to address new evidence

College Board’s Instructional Video: Understanding a Line of Reasoning .

Fiveable’s Effective Annotations .

Try Fiveable’s Guide to LOR Body Paragraphs .

🗺 Can you identify the rhetorical modes?

You won’t be asked to name drop them on the exam, but it can be helpful to use devices when discussing strategies. Try this Quizlet to help prepare.

🧐 Unit 5 Commentary and Analysis: Analysis of complex argument and intentional rhetoric

In Unit 5, the skills look at the minutiae involved in argumentation: development of the line of reasoning that produces strong commentary and maintains the primary claim through all parts of the writing. To achieve these goals, this unit includes a focus on transitions , modifiers , and qualifications for argumentative perspective .  

Unit 5 Overview

5.1 Maintaining ideas throughout an argument

5.2 Developing commentary throughout paragraphs

5.3 Using modifiers to qualify an argument and convey perspective

5.4 Using transitions

Fiveable’s video on How to Improve Analysis Part 1 and Part 2

As well as how to Embed Quotes into Body Paragraphs  

Rhetorical Analysis Body Paragraphs  

Synthesis Essay Body Paragraphs  

Argument Essay Body Paragraphs

Tara Seale’s adaptation for Creating a Line of Reasoning .

🏃‍♂️ Unit 6 Rhetorical Risks: Analysis of multiple perspectives , bias , and shifts with new evidence

In Unit 6, you will notice a direct link building on the ideas of Unit 3 as this instruction looks at position and perspectives while synthesizing information strategically to support a claim.  For greater depth, this unit moves to modify a current argument to include new evidence .

Unit 6 Overview: Position, Perspective , and Bias

6.1 Incorporating multiple perspectives strategically into an argument

6.2 Recognizing and accounting for bias

6.3 Adjusting an argument to new evidence

6.4 Analyzing tone and shifts in tone

College Board’s Instructional Video: Creating a Nuanced Argument .

Fiveable’s video on Tracking an Author’s Argument  

🚀 Unit 7 Complex Argumentation: Analysis of effective arguments, including concession and refutation

The skills of Unit 7 are about putting all units of study together to look at the complexity of a given argument and the effectiveness of the pieces built into that argument.  Though many teachers will have addressed counterarguments, concessions, and refutations before reaching this unit, those skills are highly scrutinized in this segment of learning.

Unit 7 Overview: Successful and Unsuccessful Arguments

7.1 Examining complexities in issues

7.2 Considering how words, phrases, and clauses can modify and limit an argument

7.3 Examining how counterargument or alternative perspectives affect an argument

7.4 Exploring how sentence development affects an argument

Fiveable’s video on Arguments and Counterarguments  

College Board’s Instructional Video: How Argument Demonstrates Understanding .

Check your progress with Fiveable’s AP Language Skills Matrix .

📝 Unit 8 Style: Analysis of how style influences the audience movement

Unit 8 covers how to understand the influence style has on the audience , and the purpose behind each decision. By analyzing these various tactics, students are able to understand the author’s audience , and how to effectively persuade them. Style is an important part in connecting the rest of the course and understanding how the rhetorical choices and devices are used to accomplish a purpose .

Unit 8 Overview: Stylistic Choices

8.1 Choosing comparisons based on an audience

8.2 Considering how sentence development and word choice affect how the writer is perceived by an audience

8.3 Considering how all choices made in an argument affect the audience

8.4 Considering how style affects an argument

Fiveable’s Analysis of the Mindset of the Audience

College Board’s Instructional video: Analyzing and Understanding the Audience

College Board’s explanation of Elements and Context for Style  

Review this quizlet on Elements of Style for more practice.

✏️ Unit 9 Craft: Creation of your own complex argument with synthesis and rhetoric

The final unit of AP Language and Composition covers how to effectively form your own arguments by acknowledging and understanding complexities to create a nuanced and sophisticated argument. It focuses on your ability to comprehend and connect multiple sources to create a well reasoned, and detailed argument as well as how to add in your own rhetorical devices and choices to make your writing more persuasive and effective.

Unit 9 Overview: Developing a Complex Argument

9.1 Strategically conceding, rebutting, or refuting information

9.2 Crafting an argument through stylistic choices like word choice and description

Fiveable’s video on Creating your own Synthesis Arguments

College Board’s video on Complexities within Arguments and How to Create a Nuanced Argument

Fiveable

Student Wellness

Stay connected.

© 2024 Fiveable Inc. All rights reserved.

PrepScholar

Choose Your Test

Sat / act prep online guides and tips, how to write a perfect synthesis essay for the ap language exam.

author image

Advanced Placement (AP)

body-pencil-sharpen-notebook-1

If you're planning to take the AP Language (or AP Lang) exam , you might already know that 55% of your overall exam score will be based on three essays. The first of the three essays you'll have to write on the AP Language exam is called the "synthesis essay." If you want to earn full points on this portion of the AP Lang Exam, you need to know what a synthesis essay is and what skills are assessed by the AP Lang synthesis essay.

In this article, we'll explain the different aspects of the AP Lang synthesis essay, including what skills you need to demonstrate in your synthesis essay response in order to achieve a good score. We'll also give you a full breakdown of a real AP Lang Synthesis Essay prompt, provide an analysis of an AP Lang synthesis essay example, and give you four tips for how to write a synthesis essay.

Let's get started by taking a closer look at how the AP Lang synthesis essay works!

Synthesis Essay AP Lang: What It Is and How It Works

The AP Lang synthesis essay is the first of three essays included in the Free Response section of the AP Lang exam.

The AP Lang synthesis essay portion of the Free Response section lasts for one hour total . This hour consists of a recommended 15 minute reading period and a 40 minute writing period. Keep in mind that these time allotments are merely recommendations, and that exam takers can parse out the allotted 60 minutes to complete the synthesis essay however they choose.

Now, here's what the structure of the AP Lang synthesis essay looks like. The exam presents six to seven sources that are organized around a specific topic (like alternative energy or eminent domain, which are both past synthesis exam topics).

Of these six to seven sources, at least two are visual , including at least one quantitative source (like a graph or pie chart, for example). The remaining four to five sources are print text-based, and each one contains approximately 500 words.

In addition to six to seven sources, the AP Lang exam provides a written prompt that consists of three paragraphs. The prompt will briefly explain the essay topic, then present a claim that students will respond to in an essay that synthesizes material from at least three of the sources provided.

Here's an example prompt provided by the College Board:

Directions : The following prompt is based on the accompanying six sources.

This question requires you to integrate a variety of sources into a coherent, well-written essay. Refer to the sources to support your position; avoid mere paraphrase or summary. Your argument should be central; the sources should support this argument .

Remember to attribute both direct and indirect citations.

Introduction

Television has been influential in United States presidential elections since the 1960's. But just what is this influence, and how has it affected who is elected? Has it made elections fairer and more accessible, or has it moved candidates from pursuing issues to pursuing image?

Read the following sources (including any introductory information) carefully. Then, in an essay that synthesizes at least three of the sources for support, take a position that defends, challenges, or qualifies the claim that television has had a positive impact on presidential elections.

Refer to the sources as Source A, Source B, etc.; titles are included for your convenience.

Source A (Campbell) Source B (Hart and Triece) Source C (Menand) Source D (Chart) Source E (Ranney) Source F (Koppel)

Like we mentioned earlier, this prompt gives you a topic — which it briefly explains — then asks you to take a position. In this case, you'll have to choose a stance on whether television has positively or negatively affected U.S. elections. You're also given six sources to evaluate and use in your response. Now that you have everything you need, now your job is to write an amazing synthesis essay.

But what does "synthesize" mean, exactly? According to the CollegeBoard, when an essay prompt asks you to synthesize, it means that you should "combine different perspectives from sources to form a support of a coherent position" in writing. In other words, a synthesis essay asks you to state your claim on a topic, then highlight the relationships between several sources that support your claim on that topic. Additionally, you'll need to cite specific evidence from your sources to prove your point.

The synthesis essay counts for six of the total points on the AP Lang exam . Students can receive 0-1 points for writing a thesis statement in the essay, 0-4 based on incorporation of evidence and commentary, and 0-1 points based on sophistication of thought and demonstrated complex understanding of the topic.

You'll be evaluated based on how effectively you do the following in your AP Lang synthesis essay:

Write a thesis that responds to the exam prompt with a defensible position

Provide specific evidence that to support all claims in your line of reasoning from at least three of the sources provided, and clearly and consistently explain how the evidence you include supports your line of reasoning

Demonstrate sophistication of thought by either crafting a thoughtful argument, situating the argument in a broader context, explaining the limitations of an argument

Make rhetorical choices that strengthen your argument and/or employ a vivid and persuasive style throughout your essay.

If your synthesis essay meets the criteria above, then there's a good chance you'll score well on this portion of the AP Lang exam!

If you're looking for even more information on scoring, the College Board has posted the AP Lang Free Response grading rubric on its website. ( You can find it here. ) We recommend taking a close look at it since it includes additional details about the synthesis essay scoring.

body-chisel-break-apart

Don't be intimidated...we're going to teach you how to break down even the hardest AP synthesis essay prompt.

Full Breakdown of a Real AP Lang Synthesis Essay Prompt

In this section, we'll teach you how to analyze and respond to a synthesis essay prompt in five easy steps, including suggested time frames for each step of the process.

Step 1: Analyze the Prompt

The very first thing to do when the clock starts running is read and analyze the prompt. To demonstrate how to do this, we'll look at the sample AP Lang synthesis essay prompt below. This prompt comes straight from the 2018 AP Lang exam:

Eminent domain is the power governments have to acquire property from private owners for public use. The rationale behind eminent domain is that governments have greater legal authority over lands within their dominion than do private owners. Eminent domain has been instituted in one way or another throughout the world for hundreds of years.

Carefully read the following six sources, including the introductory information for each source. Then synthesize material from at least three of the sources and incorporate it into a coherent, well-developed essay that defends, challenges, or qualifies the notion that eminent domain is productive and beneficial.

Your argument should be the focus of your essay. Use the sources to develop your argument and explain the reasoning for it. Avoid merely summarizing the sources. Indicate clearly which sources you are drawing from, whether through direct quotation, paraphrase, or summary. You may cite the sources as Source A, Source B, etc., or by using the descriptions in parentheses.

On first read, you might be nervous about how to answer this prompt...especially if you don't know what eminent domain is! But if you break the prompt down into chunks, you'll be able to figure out what the prompt is asking you to do in no time flat.

To get a full understanding of what this prompt wants you to do, you need to identify the most important details in this prompt, paragraph by paragraph. Here's what each paragraph is asking you to do:

  • Paragraph 1: The prompt presents and briefly explains the topic that you'll be writing your synthesis essay about. That topic is the concept of eminent domain.
  • Paragraph 2: The prompt presents a specific claim about the concept of eminent domain in this paragraph: Eminent domain is productive and beneficial. This paragraph instructs you to decide whether you want to defend, challenge, or qualify that claim in your synthesis essay , and use material from at least three of the sources provided in order to do so.
  • Paragraph 3: In the last paragraph of the prompt, the exam gives you clear instructions about how to approach writing your synthesis essay . First, make your argument the focus of the essay. Second, use material from at least three of the sources to develop and explain your argument. Third, provide commentary on the material you include, and provide proper citations when you incorporate quotations, paraphrases, or summaries from the sources provided.

So basically, you'll have to agree with, disagree with, or qualify the claim stated in the prompt, then use at least three sources substantiate your answer. Since you probably don't know much about eminent domain, you'll probably decide on your position after you read the provided sources.

To make good use of your time on the exam, you should spend around 2 minutes reading the prompt and making note of what it's asking you to do. That will leave you plenty of time to read the sources provided, which is the next step to writing a synthesis essay.

Step 2: Read the Sources Carefully

After you closely read the prompt and make note of the most important details, you need to read all of the sources provided. It's tempting to skip one or two sources to save time--but we recommend you don't do this. That's because you'll need a thorough understanding of the topic before you can accurately address the prompt!

For the sample exam prompt included above, there are six sources provided. We're not going to include all of the sources in this article, but you can view the six sources from this question on the 2018 AP Lang exam here . The sources include five print-text sources and one visual source, which is a cartoon.

As you read the sources, it's important to read quickly and carefully. Don't rush! Keep your pencil in hand to quickly mark important passages that you might want to use as evidence in your synthesis. While you're reading the sources and marking passages, you want to think about how the information you're reading influences your stance on the issue (in this case, eminent domain).

When you finish reading, take a few seconds to summarize, in a phrase or sentence, whether the source defends, challenges, or qualifies whether eminent domain is beneficial (which is the claim in the prompt) . Though it might not feel like you have time for this, it's important to give yourself these notes about each source so you know how you can use each one as evidence in your essay.

Here's what we mean: say you want to challenge the idea that eminent domain is useful. If you've jotted down notes about each source and what it's saying, it will be easier for you to pull the relevant information into your outline and your essay.

So how much time should you spend reading the provided sources? The AP Lang exam recommends taking 15 minutes to read the sources . If you spend around two of those minutes reading and breaking down the essay prompt, it makes sense to spend the remaining 13 minutes reading and annotating the sources.

If you finish reading and annotating early, you can always move on to drafting your synthesis essay. But make sure you're taking your time and reading carefully! It's better to use a little extra time reading and understanding the sources now so that you don't have to go back and re-read the sources later.

body-weightlifting-lift-strong

A strong thesis will do a lot of heavy lifting in your essay. (See what we did there?)

Step 3: Write a Strong Thesis Statement

After you've analyzed the prompt and thoroughly read the sources, the next thing you need to do in order to write a good synthesis essay is write a strong thesis statement .

The great news about writing a thesis statement for this synthesis essay is that you have all the tools you need to do it at your fingertips. All you have to do in order to write your thesis statement is decide what your stance is in relationship to the topic provided.

In the example prompt provided earlier, you're essentially given three choices for how to frame your thesis statement: you can either defend, challenge, or qualify a claim that's been provided by the prompt, that eminent domain is productive and beneficial . Here's what that means for each option:

If you choose to defend the claim, your job will be to prove that the claim is correct . In this case, you'll have to show that eminent domain is a good thing.

If you choose to challenge the claim, you'll argue that the claim is incorrect. In other words, you'll argue that eminent domain isn't productive or beneficial.

If you choose to qualify, that means you'll agree with part of the claim, but disagree with another part of the claim. For instance, you may argue that eminent domain can be a productive tool for governments, but it's not beneficial for property owners. Or maybe you argue that eminent domain is useful in certain circumstances, but not in others.

When you decide whether you want your synthesis essay to defend, challenge, or qualify that claim, you need to convey that stance clearly in your thesis statement. You want to avoid simply restating the claim provided in the prompt, summarizing the issue without making a coherent claim, or writing a thesis that doesn't respond to the prompt.

Here's an example of a thesis statement that received full points on the eminent domain synthesis essay:

Although eminent domain can be misused to benefit private interests at the expense of citizens, it is a vital tool of any government that intends to have any influence on the land it governs beyond that of written law.

This thesis statement received full points because it states a defensible position and establishes a line of reasoning on the issue of eminent domain. It states the author's position (that some parts of eminent domain are good, but others are bad), then goes on to explain why the author thinks that (it's good because it allows the government to do its job, but it's bad because the government can misuse its power.)

Because this example thesis statement states a defensible position and establishes a line of reasoning, it can be elaborated upon in the body of the essay through sub-claims, supporting evidence, and commentary. And a solid argument is key to getting a six on your synthesis essay for AP Lang!

Looking for help studying for your AP exam?

Our one-on-one online AP tutoring services can help you prepare for your AP exams. Get matched with a top tutor who got a high score on the exam you're studying for!

Get a 5 On Your AP Exam

Step 4: Create a Bare-Bones Essay Outline

Once you've got your thesis statement drafted, you have the foundation you need to develop a bare bones outline for your synthesis essay. Developing an outline might seem like it's a waste of your precious time, but if you develop your outline well, it will actually save you time when you start writing your essay.

With that in mind, we recommend spending 5 to 10 minutes outlining your synthesis essay . If you use a bare-bones outline like the one below, labeling each piece of content that you need to include in your essay draft, you should be able to develop out the most important pieces of the synthesis before you even draft the actual essay.

To help you see how this can work on test day, we've created a sample outline for you. You can even memorize this outline to help you out on test day! In the outline below, you'll find places to fill in a thesis statement, body paragraph topic sentences, evidence from the sources provided, and commentary :

  • Present the context surrounding the essay topic in a couple of sentences (this is a good place to use what you learned about the major opinions or controversies about the topic from reading your sources).
  • Write a straightforward, clear, and concise thesis statement that presents your stance on the topic
  • Topic sentence presenting first supporting point or claim
  • Evidence #1
  • Commentary on Evidence #1
  • Evidence #2 (if needed)
  • Commentary on Evidence #2 (if needed)
  • Topic sentence presenting second supporting point or claim
  • Topic sentence presenting three supporting point or claim
  • Sums up the main line of reasoning that you developed and defended throughout the essay
  • Reiterates the thesis statement

Taking the time to develop these crucial pieces of the synthesis in a bare-bones outline will give you a map for your final essay. Once you have a map, writing the essay will be much easier.

Step 5: Draft Your Essay Response

The great thing about taking a few minutes to develop an outline is that you can develop it out into your essay draft. After you take about 5 to 10 minutes to outline your synthesis essay, you can use the remaining 30 to 35 minutes to draft your essay and review it.

Since you'll outline your essay before you start drafting, writing the essay should be pretty straightforward. You'll already know how many paragraphs you're going to write, what the topic of each paragraph will be, and what quotations, paraphrases, or summaries you're going to include in each paragraph from the sources provided. You'll just have to fill in one of the most important parts of your synthesis—your commentary.

Commentaries are your explanation of why your evidence supports the argument you've outlined in your thesis. Your commentary is where you actually make your argument, which is why it's such a critical part of your synthesis essay.

When thinking about what to say in your commentary, remember one thing the AP Lang synthesis essay prompt specifies: don't just summarize the sources. Instead, as you provide commentary on the evidence you incorporate, you need to explain how that evidence supports or undermines your thesis statement . You should include commentary that offers a thoughtful or novel perspective on the evidence from your sources to develop your argument.

One very important thing to remember as you draft out your essay is to cite your sources. The AP Lang exam synthesis essay prompt indicates that you can use generic labels for the sources provided (e.g. "Source 1," "Source 2," "Source 3," etc.). The exam prompt will indicate which label corresponds with which source, so you'll need to make sure you pay attention and cite sources accurately. You can cite your sources in the sentence where you introduce a quote, summary, or paraphrase, or you can use a parenthetical citation. Citing your sources affects your score on the synthesis essay, so remembering to do this is important.

body-green-arrow-down

Keep reading for a real-life example of a great AP synthesis essay response!

Real-Life AP Synthesis Essay Example and Analysis

If you're still wondering how to write a synthesis essay, examples of real essays from past AP Lang exams can make things clearer. These real-life student AP synthesis essay responses can be great for helping you understand how to write a synthesis essay that will knock the graders' socks off .

While there are multiple essay examples online, we've chosen one to take a closer look at. We're going to give you a brief analysis of one of these example student synthesis essays from the 2019 AP Lang Exam below!

Example Synthesis Essay AP Lang Response

To get started, let's look at the official prompt for the 2019 synthesis essay:

In response to our society's increasing demand for energy, large-scale wind power has drawn attention from governments and consumers as a potential alternative to traditional materials that fuel our power grids, such as coal, oil, natural gas, water, or even newer sources such as nuclear or solar power. Yet the establishment of large-scale, commercial-grade wind farms is often the subject of controversy for a variety of reasons.

Carefully read the six sources, found on the AP English Language and Composition 2019 Exam (Question 1), including the introductory information for each source. Write an essay that synthesizes material from at least three of the sources and develops your position on the most important factors that an individual or agency should consider when deciding whether to establish a wind farm.

Source A (photo) Source B (Layton) Source C (Seltenrich) Source D (Brown) Source E (Rule) Source F (Molla)

In your response you should do the following:

  • Respond to the prompt with a thesis presents a defensible position.
  • Select and use evidence from at least 3 of the provided sources to support your line of reasoning. Indicate clearly the sources used through direct quotation, paraphrase, or summary. Sources may be cited as Source A, Source B, etc., or by using the description in parentheses.
  • Explain how the evidence supports your line of reasoning.
  • Use appropriate grammar and punctuation in communicating your argument.

Now that you know exactly what the prompt asked students to do on the 2019 AP Lang synthesis essay, here's an AP Lang synthesis essay example, written by a real student on the AP Lang exam in 2019:

[1] The situation has been known for years, and still very little is being done: alternative power is the only way to reliably power the changing world. The draw of power coming from industry and private life is overwhelming current sources of non-renewable power, and with dwindling supplies of fossil fuels, it is merely a matter of time before coal and gas fuel plants are no longer in operation. So one viable alternative is wind power. But as with all things, there are pros and cons. The main factors for power companies to consider when building wind farms are environmental boon, aesthetic, and economic factors.

[2] The environmental benefits of using wind power are well-known and proven. Wind power is, as qualified by Source B, undeniably clean and renewable. From their production requiring very little in the way of dangerous materials to their lack of fuel, besides that which occurs naturally, wind power is by far one of the least environmentally impactful sources of power available. In addition, wind power by way of gearbox and advanced blade materials, has the highest percentage of energy retention. According to Source F, wind power retains 1,164% of the energy put into the system – meaning that it increases the energy converted from fuel (wind) to electricity 10 times! No other method of electricity production is even half that efficient. The efficiency and clean nature of wind power are important to consider, especially because they contribute back to power companies economically.

[3] Economically, wind power is both a boon and a bone to electric companies and other users. For consumers, wind power is very cheap, leading to lower bills than from any other source. Consumers also get an indirect reimbursement by way of taxes (Source D). In one Texan town, McCamey, tax revenue increased 30% from a wind farm being erected in the town. This helps to finance improvements to the town. But, there is no doubt that wind power is also hurting the power companies. Although, as renewable power goes, wind is incredibly cheap, it is still significantly more expensive than fossil fuels. So, while it is helping to cut down on emissions, it costs electric companies more than traditional fossil fuel plants. While the general economic trend is positive, there are some setbacks which must be overcome before wind power can take over as truly more effective than fossil fuels.

[4] Aesthetics may be the greatest setback for power companies. Although there may be significant economic and environmental benefit to wind power, people will always fight to preserve pure, unspoiled land. Unfortunately, not much can be done to improve the visual aesthetics of the turbines. White paint is the most common choice because it "[is] associated with cleanliness." (Source E). But, this can make it stand out like a sore thumb, and make the gargantuan machines seem more out of place. The site can also not be altered because it affects generating capacity. Sound is almost worse of a concern because it interrupts personal productivity by interrupting people's sleep patterns. One thing for power companies to consider is working with turbine manufacturing to make the machines less aesthetically impactful, so as to garner greater public support.

[5] As with most things, wind power has no easy answer. It is the responsibility of the companies building them to weigh the benefits and the consequences. But, by balancing economics, efficiency, and aesthetics, power companies can create a solution which balances human impact with environmental preservation.

And that's an entire AP Lang synthesis essay example, written in response to a real AP Lang exam prompt! It's important to remember AP Lang exam synthesis essay prompts are always similarly structured and worded, and students often respond in around the same number of paragraphs as what you see in the example essay response above.

Next, let's analyze this example essay and talk about what it does effectively, where it could be improved upon, and what score past exam scorers awarded it.

To get started on an analysis of the sample synthesis essay, let's look at the scoring commentary provided by the College Board:

  • For development of thesis, the essay received 1 out of 1 possible points
  • For evidence and commentary, the essay received 4 out of 4 possible points
  • For sophistication of thought, the essay received 0 out of 1 possible points.

This means that the final score for this example essay was a 5 out of 6 possible points . Let's look more closely at the content of the example essay to figure out why it received this score breakdown.

Thesis Development

The thesis statement is one of the three main categories that is taken into consideration when you're awarded points on this portion of the exam. This sample essay received 1 out of 1 total points.

Now, here's why: the thesis statement clearly and concisely conveys a position on the topic presented in the prompt--alternative energy and wind power--and defines the most important factors that power companies should consider when deciding whether to establish a wind farm.

Evidence and Commentary

The second key category taken into consideration when synthesis exams are evaluated is incorporation of evidence and commentary. This sample received 4 out of 4 possible points for this portion of the synthesis essay. At bare minimum, this sample essay meets the requirement mentioned in the prompt that the writer incorporate evidence from at least three of the sources provided.

On top of that, the writer does a good job of connecting the incorporated evidence back to the claim made in the thesis statement through effective commentary. The commentary in this sample essay is effective because it goes beyond just summarizing what the provided sources say. Instead, it explains and analyzes the evidence presented in the selected sources and connects them back to supporting points the writer makes in each body paragraph.

Finally, the writer of the essay also received points for evidence and commentary because the writer developed and supported a consistent line of reasoning throughout the essay . This line of reasoning is summed up in the fourth paragraph in the following sentence: "One thing for power companies to consider is working with turbine manufacturing to make the machines less aesthetically impactful, so as to garner greater public support."

Because the writer did a good job consistently developing their argument and incorporating evidence, they received full marks in this category. So far, so good!

Sophistication of Thought

Now, we know that this essay received a score of 5 out of 6 total points, and the place where the writer lost a point was on the basis of sophistication of thought, for which the writer received 0 out of 1 points. That's because this sample essay makes several generalizations and vague claims where it could have instead made specific claims that support a more balanced argument.

For example, in the following sentence from the 5th paragraph of the sample essay, the writer misses the opportunity to state specific possibilities that power companies should consider for wind energy . Instead, the writer is ambiguous and non-committal, saying, "As with most things, wind power has no easy answer. It is the responsibility of the companies building them to weigh the benefits and consequences."

If the writer of this essay was interested in trying to get that 6th point on the synthesis essay response, they could consider making more specific claims. For instance, they could state the specific benefits and consequences power companies should consider when deciding whether to establish a wind farm. These could include things like environmental impacts, economic impacts, or even population density!

Despite losing one point in the last category, this example synthesis essay is a strong one. It's well-developed, thoughtfully written, and advances an argument on the exam topic using evidence and support throughout.

body-number-four-post-it-note

4 Tips for How to Write a Synthesis Essay

AP Lang is a timed exam, so you have to pick and choose what you want to focus on in the limited time you're given to write the synthesis essay. Keep reading to get our expert advice on what you should focus on during your exam.

Tip 1: Read the Prompt First

It may sound obvious, but when you're pressed for time, it's easy to get flustered. Just remember: when it comes time to write the synthesis essay, read the prompt first !

Why is it so important to read the prompt before you read the sources? Because when you're aware of what kind of question you're trying to answer, you'll be able to read the sources more strategically. The prompt will help give you a sense of what claims, points, facts, or opinions to be looking for as you read the sources.

Reading the sources without having read the prompt first is kind of like trying to drive while wearing a blindfold: you can probably do it, but it's likely not going to end well!

Tip 2: Make Notes While You Read

During the 15-minute reading period at the beginning of the synthesis essay, you'll be reading through the sources as quickly as you can. After all, you're probably anxious to start writing!

While it's definitely important to make good use of your time, it's also important to read closely enough that you understand your sources. Careful reading will allow you to identify parts of the sources that will help you support your thesis statement in your essay, too.

As you read the sources, consider marking helpful passages with a star or check mark in the margins of the exam so you know which parts of the text to quickly re-read as you form your synthesis essay. You might also consider summing up the key points or position of each source in a sentence or a few words when you finish reading each source during the reading period. Doing so will help you know where each source stands on the topic given and help you pick the three (or more!) that will bolster your synthesis argument.

Tip 3: Start With the Thesis Statement

If you don't start your synthesis essay with a strong thesis statement, it's going to be tough to write an effective synthesis essay. As soon as you finish reading and annotating the provided sources, the thing you want to do next is write a strong thesis statement.

According to the CollegeBoard grading guidelines for the AP Lang synthesis essay, a strong thesis statement will respond to the prompt— not restate or rephrase the prompt. A good thesis will take a clear, defensible position on the topic presented in the prompt and the sources.

In other words, to write a solid thesis statement to guide the rest of your synthesis essay, you need to think about your position on the topic at hand and then make a claim about the topic based on your position. This position will either be defending, challenging, or qualifying the claim made in the essay's prompt.

The defensible position that you establish in your thesis statement will guide your argument in the rest of the essay, so it's important to do this first. Once you have a strong thesis statement, you can begin outlining your essay.

Tip 4: Focus on Your Commentary

Writing thoughtful, original commentary that explains your argument and your sources is important. In fact, doing this well will earn you four points (out of a total of six)!

AP Lang provides six to seven sources for you on the exam, and you'll be expected to incorporate quotations, paraphrases, or summaries from at least three of those sources into your synthesis essay and interpret that evidence for the reader.

While incorporating evidence is very important, in order to get the extra point for "sophistication of thought" on the synthesis essay, it's important to spend more time thinking about your commentary on the evidence you choose to incorporate. The commentary is your chance to show original thinking, strong rhetorical skills, and clearly explain how the evidence you've included supports the stance you laid out in your thesis statement.

To earn the 6th possible point on the synthesis essay, make sure your commentary demonstrates a nuanced understanding of the source material, explains this nuanced understanding, and places the evidence incorporated from the sources in conversation with each other. To do this, make sure you're avoiding vague language. Be specific when you can, and always tie your commentary back to your thesis!

body-person-arrows-next

What's Next?

There's a lot more to the AP Language exam than just the synthesis essay. Be sure to check out our expert guide to the entire exam , then learn more about the tricky multiple choice section .

Is the AP Lang exam hard...or is it easy? See how it stacks up to other AP tests on our list of the hardest AP exams .

Did you know there are technically two English AP exams? You can learn more about the second English AP test, the AP Literature exam, in this article . And if you're confused about whether you should take the AP Lang or AP Lit test , we can help you make that decision, too.

Want to improve your SAT score by 160 points or your ACT score by 4 points? We've written a guide for each test about the top 5 strategies you must be using to have a shot at improving your score. Download it for free now:

Get eBook: 5 Tips for 160+ Points

Ashley Sufflé Robinson has a Ph.D. in 19th Century English Literature. As a content writer for PrepScholar, Ashley is passionate about giving college-bound students the in-depth information they need to get into the school of their dreams.

Student and Parent Forum

Our new student and parent forum, at ExpertHub.PrepScholar.com , allow you to interact with your peers and the PrepScholar staff. See how other students and parents are navigating high school, college, and the college admissions process. Ask questions; get answers.

Join the Conversation

Ask a Question Below

Have any questions about this article or other topics? Ask below and we'll reply!

Improve With Our Famous Guides

  • For All Students

The 5 Strategies You Must Be Using to Improve 160+ SAT Points

How to Get a Perfect 1600, by a Perfect Scorer

Series: How to Get 800 on Each SAT Section:

Score 800 on SAT Math

Score 800 on SAT Reading

Score 800 on SAT Writing

Series: How to Get to 600 on Each SAT Section:

Score 600 on SAT Math

Score 600 on SAT Reading

Score 600 on SAT Writing

Free Complete Official SAT Practice Tests

What SAT Target Score Should You Be Aiming For?

15 Strategies to Improve Your SAT Essay

The 5 Strategies You Must Be Using to Improve 4+ ACT Points

How to Get a Perfect 36 ACT, by a Perfect Scorer

Series: How to Get 36 on Each ACT Section:

36 on ACT English

36 on ACT Math

36 on ACT Reading

36 on ACT Science

Series: How to Get to 24 on Each ACT Section:

24 on ACT English

24 on ACT Math

24 on ACT Reading

24 on ACT Science

What ACT target score should you be aiming for?

ACT Vocabulary You Must Know

ACT Writing: 15 Tips to Raise Your Essay Score

How to Get Into Harvard and the Ivy League

How to Get a Perfect 4.0 GPA

How to Write an Amazing College Essay

What Exactly Are Colleges Looking For?

Is the ACT easier than the SAT? A Comprehensive Guide

Should you retake your SAT or ACT?

When should you take the SAT or ACT?

Stay Informed

ap lang argumentative essay

Get the latest articles and test prep tips!

Looking for Graduate School Test Prep?

Check out our top-rated graduate blogs here:

GRE Online Prep Blog

GMAT Online Prep Blog

TOEFL Online Prep Blog

Holly R. "I am absolutely overjoyed and cannot thank you enough for helping me!”

404 Not found

Get in touch with us

Are you sure you want to logout?

Study abroad.

bannerAd

AP Lang Rhetorical Analysis Essay

AP language exams are held for numerous subjects. Students have to choose a specific subject and get higher scores. The higher the score, the higher the chance of pursuing the best 700 colleges or universities overall in the world, including the United States and Canada.

When it comes to AP English language, it involves a section called rhetorical analysis essay. This is a part of three free-response essays that have to be answered within 2 hours and 15 minutes from the overall 3 hours 15 minutes exam. 

parallel

If you are taking the AP Lang exam this year, guidance on how to answer this part will be useful. This article is specially curated to help you score the best. Read on to learn more about the AP language rhetorical analysis essay and get an idea of how to prepare for the associated exam successfully.

What is the AP Lang Rhetorical Analysis Essay ?

AP elaborated that the Advanced Placement is the exam conducted by the College Board in the United States of America. The exam is generally offered at the high school level and helps students to pursue higher education at the university level. The exam holds two sections: MCQs and Rhetorical Essay. MCQ sections are for 1 hour, and the rest of the time is for the essay section. The free-response essay holds three essays: rhetorical analysis essay, synthesis essay, and argumentative essay. 

parallel

  • The rhetorical analysis essay in the AP Lang exam involves students having to discuss how the authors’ contribution to the passage gives a theme or meaning. 
  • A synthesis essay involves students creating arguments on the passage or piece of information delivered to them.
  • An argumentative essay requires students to pick a side ‘for or against ‘ for an argument or debate.

Since we are here to discuss rhetorical analysis essays in AP language, you must know the essay is added to test students’ ability to analyze and interpret the deeper meaning in the provided passage. Through rhetorical essays, the examiner examines how students connect with the author’s style of writing and syntax within 40 minutes. Some students may face challenges while dealing with this essay because it requires a better understanding of rhetorical strategies and the method to apply them.

Tips to Write a Rhetorical Analysis Essay AP Lang ?

Since a rhetorical essay is quite tricky in comparison to other essays, it requires certain tips for a better approach to answering. Let us explore the method of writing a rhetorical essay for the AP Lang exam that helps you score well.

parallel

  • Outline Essay Prior to Writing

Not only good content but providing a readable structure is an important part of the rhetorical analysis essay AP Lang . First, you must read the passage thoroughly and develop a brief outline or key points before writing the essay. This helps you write with respect to the chronology of the given passage and maintain the flow of writing.

  • Understand Rhetorical Strategies

You aren’t alone in thinking about where to start writing a rhetorical essay. Many students face the same and are required to learn and implement rhetorical strategies. First, understand what rhetoric actually means. It refers to language sensibly chosen and structured for an impressive effect on the audience. This involves persuasive appeal, logical fallacies, and syntax such as anaphora, anthesis, parallelism, and so on. There is a vast range of elements that you can assess in the provided essay and develop a strong grasp with consistent practice.

parallel

  • Make your Essay Well-Structured

Sometimes, students understand the rhetorical strategies but still get confused about where to initiate. It is recommended to start with an introduction that delivers the purpose of your writing. In the last introductory line, you must talk about the rhetorical strategies you will discuss in the piece. However, there are so many styles, syntax, and tones. You must be specific while listing them and then move to develop a body paragraph. 

Now, you have to collect all the rhetorical strategies you mentioned in the introduction to discuss your point of view chronologically. Be specific while discussing strategy, as only the crucial ones among them must be discussed. Never forget to cite the line from the original passage. Your write-up must maintain the flow and should include relativity among the paragraphs. End the rhetorical essay by summarising key points.

parallel

  • Never Forget to Explain your Examples

Do not just state the examples or deliver statements like ‘this is an example of pathos or logos.’ It is advisable to explain the example you have listed in context to the rhetorical elements you have mentioned and how it aids the author in their viewpoint. Stay detailed yet precise while writing the rhetorical analysis essay AP Lang .

AP Lang Rhetorical Analysis Essay Rubric

Did you know that the rhetorical analysis essay AP Lang is graded into three rubric categories? Students must pay heed to the categories, as the examiner seeks specific things in each of them. Also, learn about some dos and don’ts to score well.

parallel

When it comes to grading the thesis of the rhetorical analysis essay AP Lang , there is nothing nebulous. Either you get one point, or you lose one. Thesis points help you get close to higher scores, and thus, you must be mindful of the following points.

parallel

  • Developing an argument states that your interpretation leads to a risk of disagreement. Thus, your thesis statements must be in context with the author’s rhetorical choice. 
  • If you create a phrase in your mind initiating with ‘I think that..’, your phrase should not go in a negative argumentative direction.
  • Never provide a thesis with summaries but not an argument.
  • Try not to provide the thesis with repeated prompts.

Evidence is provided to prove the argumentative context in the thesis. This rubric category is graded from 0 to 4 points. To score higher, you must follow the points below.

  • You must aim to provide multiple types of evidence in your argumentative thesis.
  • Each statement you provide must be backed up with evidence in context to the text or arguments about the author’s theoretical choices.
  • Add more and more evidence, which must be specific.
  • Examine whether your evidence is linked with your overarching argument.
  • Deliver your interpretation and never rely on just quotes or phrases. 
  • Steer clear of generalization for text or author
  • Avoid quotes that speak for themselves. You must elaborate on the evidence you provided.
  • Sophistication

According to The College Board, this category holds 0 to 1 points. The higher grading in rhetorical analysis essays indicates the delivery of the sophistication of thoughts or a complex understanding of rhetorical elements. In this, you have to mainly focus on the number of semicolons you use and not the fancy terms. Here is what you need to follow:

  • Focus on delivering the right connection between the thesis and your evidence.
  • Create a brief framework and then proceed with writing the essay in the right flow to stay precise and clear in your piece.
  • Do not include arguments that you won’t be able to provide evidence for.
  • Ignore complex or fancy words or phrases that are hard to follow.

Ways to Improve Your Rhetorical Analysis Essay AP Lang

Now that you know what a rhetorical analysis essay is and how it is developed, you must be thinking about how tricky it is to understand the passage. Many find themselves in a completely blank position when it comes to where to begin during the exam. They find the rhetorical analysis essay AP Lang quite challenging to deal with and understand the author’s perspective and viewpoint. Practicing rhetorical strategies does not help if you do not have the right approach, which only comes from the field experts and their guidance.  Turito offers a platform where you get mentors to teach you all the concepts and approaches you apply in writing effective rhetorical essays in less time. Their course helps you get higher scores, which you can use to get college admission to your favorite college and country. Hey, you ambitious one, what are you waiting for? Contact us now and give your dreams a flight!

Frequently Asked Questions

What courses do turito offer.

Turito offers several courses to help you prepare. Here are some of them for your reference. 1. Foundation Course 2. IIT JEE 3. NEET 4. Study Abroad 5. Universal Program 6. PSAT 7. AP 8. IELTS

Can I ask queries on Turito?

Yes absolutely! Turito offers a platform where you can ask your concerns and queries. You will get answers from the field expert that can let your worries fade away.

Is Turito genuine?

Turito is students’ most trusted learning platform, among others. Turito believes in transparency and thus provides genuine feedback from students and parents on the official website. At Turito, we offer guaranteed success for hardworking students. Thus, there isn’t any negative feedback to hide. If you have any queries, you can write to us at [email protected]. Our experts will connect and assist you in no time.

AP Lang Rhetorical Analysis Essay

Relevant Articles

AP Lang Synthesis Essay

Steps to Draft AP Lang Synthesis Essay

The synthesis essay AP lang is part of the AP …

Steps to Draft AP Lang Synthesis Essay Read More »

AP Calculus AB Exam

Strategies for Success: A Complete Overview of the AP Calculus AB Exam

The AP Calculus AB Exam serves as an important element …

Strategies for Success: A Complete Overview of the AP Calculus AB Exam Read More »

Digital AP Exams

Preparing for Success: A Guide to the Features of Digital AP Exams

Digital  Advanced Placement (AP) exams bring about a huge change …

Preparing for Success: A Guide to the Features of Digital AP Exams Read More »

card img

With Turito Study Abroad

card img

Get an Expert Advice from Turito

card img

With Turito CAP.

card img

With Turito Coding.

card img

With Turito RoboNinja

card img

1-on-1 tutoring for the undivided attention

IMAGES

  1. AP Language and Composition argumentative essay

    ap lang argumentative essay

  2. How To Write An AP Lang Argumentative Essay?

    ap lang argumentative essay

  3. How to Get a 6 on Argument FRQ in AP® English Language

    ap lang argumentative essay

  4. AmStud

    ap lang argumentative essay

  5. How To Write Conclusion In Argumentative Essay Ap Lang

    ap lang argumentative essay

  6. How to Get a 6 on Argument FRQ in AP® English Language

    ap lang argumentative essay

VIDEO

  1. What is the article "an"?

  2. AP Lang Fallacies Project

  3. CSEC English A: Argumentative Writing

  4. AP Lang Rhetorical Analysis Essay Review and Practice

  5. ARGUMENTATIVE ESSAY 2024 I AP Spanish Language and Culture Exam

  6. Argumentative essay examples I Essay writing online

COMMENTS

  1. How to Write the AP Lang Argument Essay + Examples

    What Is the AP Language Argument Essay? The AP English Language Exam as of 2023 is structured as follows: Section 1: 45 multiple choice questions to be completed in an hour. This portion counts for 45% of your score. This section requires students to analyze a piece of literature.

  2. How to Write the AP Lang Argument Essay (With Example)

    Ap Lang Argumentative Essay - Expert advice on how to pen a winning essay + an AP Lang argument essay example to guide your writing.

  3. AP English Language and Composition Past Exam Questions

    Free-Response Questions. Download free-response questions from past exams along with scoring guidelines, sample responses from exam takers, and scoring distributions. If you are using assistive technology and need help accessing these PDFs in another format, contact Services for Students with Disabilities at 212-713-8333 or by email at ssd@info ...

  4. How to Get a 6 on Argument FRQ in AP® English Language

    Here's an example from the 2019 AP® English Language argumentative essay. Though there are just two short paragraphs, there is a lot of room for confusion here. In this case, "Then, write a well-developed essay in which you explain your judgment." is the key sentence you are looking for.

  5. PDF AP English Language and Composition

    AP English Language and Composition Student Samples from the 2023 Exam Administration Argument Essay 6 points

  6. PDF AP English Language and Composition Question 3: Argument (2019) Sample

    AP English Language and Composition 2019 FRQ 3 Sample Student Responses - Scored with the 2020 Rubrics The student responses in this packet were selected from the 2019 Reading and have been rescored using the new rubrics for 2020. Commentaries for each sample are provided in a separate document.

  7. PDF AP English Language and Composition

    AP® English Language and Composition 2021 Scoring Commentary Question 3 Note: Student samples are quoted verbatim and may contain spelling and grammatical errors. Overview The argument prompt for this year's exam asked students to write an essay that argued their position on the value of striving for perfection.

  8. How to Ace the AP Language Argument Essay

    3.1K 163K views 3 years ago AP Language and Composition Learn how to write the AP Language & Composition argument essay step by step in this tutorial! I'll show you the prompt from the...

  9. Crafting an Impressive Argumentative Essay for AP Lang

    Writing an argumentative essay for AP Language and Composition requires a strategic approach to effectively convey your perspective. Here's a guide to crafting an impressive argumentative essay: 1. Understand the Prompt: - Tip: Carefully read and analyze the prompt.

  10. How to Craft an Argument for AP® English Language

    The AP® English Language persuasive (or argumentative) essay is one of the three long-form free-response questions that will make up 55% of your score on the AP® English Language and Composition Exam.

  11. PDF ap06 english lang student samples

    AP® ENGLISH LANGUAGE AND COMPOSITION 2006 SCORING GUIDELINES Question 3 The score should reflect a judgment of the quality of the essay as a whole. Students had only 40 minutes to read and write; the essay, therefore, is not a finished product and should not be judged by standards appropriate for an out-of-class assignment.

  12. PDF AP ENGLISH LANGUAGE AND COMPOSITION 2016 SCORING GUIDELINES

    1 - Essays earning a score of 1 meet the criteria for the score of 2 but are undeveloped, especially simplistic in their explanation and argument, weak in their control of language, or especially lacking in coherence and development.

  13. Mastering The Argumentative Essay In AP Lang: Strategies And Examples

    Crafting an AP Lang argumentative essay requires a combination of skills, strategies, and practice. By understanding the prompt, developing a clear thesis, providing relevant evidence, and utilizing persuasive techniques, students can create compelling essays. It is important to allocate enough time for preparation, revision, and editing to ...

  14. AP Lang Argument Essay: Evidence

    9 min read • october 30, 2020 Stephanie Kirk REHUGO: We aren't sure where it started, but many teachers use REHUGO to help students find evidence on the Argument FRQ This acronym provides a quick check that can help you build logical evidence that supports your claim . R - Reading

  15. PDF AP English Language and Composition

    AP® English Language and Composition 2022 Scoring Guidelines ... Carefully read the following six sources, including the introductory information for each source. Write an essay that synthesizes material from at least ... • Use appropriate grammar and punctuation in communicating your argument. AP® English Language and Composition 2022 ...

  16. Expert Guide to the AP Language and Composition Exam

    Essay Three: Argument. In the third essay, you will be presented with an issue and asked to write a persuasive essay taking a position on the issue. You will need to support your position with evidence from your "reading, experience, and observations." Example (from 2022 free response questions): This doesn't look like a very well-constructed ...

  17. AP Lang Exam Guide

    1 argument essay. Scoring Rubric for the 2024 AP Lang Essays. Synthesis Essay. Thesis. 1 point for a defensible thesis that responds to the prompt. Evidence and Commentary. Max of 4 points for providing evidence from at least 3 sources that support the line of reasoning AND commentary that explains and analyzes the evidence. Sophistication

  18. PDF AP English Language and Composition 2018 FRQ 3 Sample Student Responses

    AP English Language and Composition Question 3: Argument (2018) Sample Student Responses 3 Sample G [1] When I was nine year sold, my parents celebrated my birthday by taking me to a ropes course. As we waited with our group to climb poles, cross logs, and hang from ropes, I began to get cold feet. I've never done this before, I thought.

  19. PDF AP English Language and Composition

    AP® English Language and Composition 2022 Scoring Guidelines. Argument Essay 6 points . Colin Powell, a four-star general and former United States secretary of state, wrote in his 1995 autobiography: "[W]e do not have the luxury of collecting information indefinitely. At some point, before we can have every possible fact in hand, we have to ...

  20. How to Write a Perfect Synthesis Essay for the AP Language Exam

    If you're planning to take the AP Language (or AP Lang) exam, you might already know that 55% of your overall exam score will be based on three essays.The first of the three essays you'll have to write on the AP Language exam is called the "synthesis essay." If you want to earn full points on this portion of the AP Lang Exam, you need to know what a synthesis essay is and what skills are ...

  21. How to Write who AP Lang Argument Essay + Examples

    The AP English Language Exam as of 2023 is structured as next: Section 1: 45 multi-user choice questions to be completed within an hour. Like parcel counts for 45% of our score. This section requires students to analyze a piece of literature. The questions ask about its site and/or what could be edited at the passage.

  22. AP Lang Rhetorical Analysis Essay: Tips & Rubric

    The rhetorical analysis essay in the AP Lang exam involves students having to discuss how the authors' contribution to the passage gives a theme or meaning. A synthesis essay involves students creating arguments on the passage or piece of information delivered to them. An argumentative essay requires students to pick a side 'for or against ...

  23. PDF AP® English Language

    1. Crafting a nuanced argument by consistently identifying and exploring complexities or tensions across the sources. 2. Articulating the implications or limitations of an argument (either the student's argument or arguments conveyed in the sources) by situating it within a broader context. 3.

  24. AP English Language and Composition

    Argument: Students will create an evidence-based argument that responds to a given topic. Exam Questions and Scoring Information 2023: Free-Response Questions Past Exam Questions and Scoring Information Article AP English Language and Composition Past Exam Questions